LSAT and Law School Admissions Forum

Get expert LSAT preparation and law school admissions advice from PowerScore Test Preparation.

User avatar
 Dave Killoran
PowerScore Staff
  • PowerScore Staff
  • Posts: 5853
  • Joined: Mar 25, 2011
|
#46267
Complete Question Explanation
(The complete setup for this game can be found here: lsat/viewtopic.php?t=14920)

The correct answer choice is (A)

Answer choice (A) is the correct answer choice, because it is the only viable setup among the choices provided:

Answer choice (B) is incorrect, because if F is on team 2, then team 1 must have M, since the team needs an anthropologist and J is to remain unassigned. With M on one team, and F on the other, S is prohibited from both teams; without J and S, there cannot be enough researchers to complete the two teams.

Answer choice (C) is incorrect, as we should recognize from question #16: F and M cannot be together on team 2.

Answer choice (D) can be ruled out, because if J is on team 1, R must be on team 2. But team 2 also needs an anthropologist; if F must be unassigned, this leaves M as the only option—but M cannot be placed on the same team as R.

Answer choice (E) cannot be true, because J on team 1 forces R to team 2, and the rules prohibit R and N from being placed on the same team.
 VamosRafa19
  • Posts: 39
  • Joined: Nov 14, 2020
|
#81685
What's the best way to approach a question like this? Assuming we see that C can be eliminated with the previous answer, there's still four viable answers not easily eliminated with any not laws? Just start applying rules and see what happens? That can take a long time, especially if doing hypotheticals.
 Robert Carroll
PowerScore Staff
  • PowerScore Staff
  • Posts: 1787
  • Joined: Dec 06, 2013
|
#81691
Vamos,

After getting rid of answer choice (C) because it's something we already know can't happen, I would also look at any previous work doing the questions and see if there's a diagram I can readily modify to make an answer work. Otherwise, I'd start checking the answers that are "easiest" to test. Although you're right that that can be time-consuming, I don't think it's that bad for some of these answers. For instance, answer choices (B) and (D) both say that someone is "not on any team". Well, the game has 7 people, 6 of whom are selected. So if 1 person is not selected, that forces everyone else in. There will be many pairs of people forced in who can't be together because of the rules, so you can start splitting those pairs between the two groups. The diagram should start filling up pretty quickly, which makes those answers more straightforward to test.

Answer choice (E) has Jones on team 1, which is a big deal - that triggers the conditional in the last rule. I can use that and the second rule to eliminate the answer, and now I have only answer choice (A) left.

Robert Carroll
User avatar
 alyosha99
  • Posts: 2
  • Joined: May 25, 2021
|
#88094
Another easy way of doing this is to notice that the answer to question 12 is C, and it has F playing for 1 and N playing for 2

Get the most out of your LSAT Prep Plus subscription.

Analyze and track your performance with our Testing and Analytics Package.